Search found 164 matches


Hi :) I like the way the problem was explained by the expert. Moreover, it is worth to remember about the roles of antecedent and consequent of a conditional. The antecedent of a conditional is a sufficient condition for the consequent. The consequent of a conditional is a necessary condition for th...

by Maciek

Fri Aug 19, 2011 6:15 am
Forum: Problem Solving
Topic: Im still confused abt this question
Replies: 4
Views: 1515

Hi all! I choose original answer. It is D. It is parallel: "San Antonio, Texas, has...than any city does" Answer C is also parallel, but has completely different meaning. Answers A, B and E are not parallel. Hope it helps! If you don't agree with my reasoning, please post here. Thanks! Bes...

by Maciek

Fri Sep 24, 2010 3:04 am
Forum: Sentence Correction
Topic: one more.
Replies: 17
Views: 3314

Hi! IMO B The main issues here are parallelism, wordiness and use of 'which'/'that' 1. Parallelism (A) 'which are harmless' - this is additional information, let us skip it, and check for parallelism. "...that... and which..." this structure is not parallel (C) "...that... and which.....

by Maciek

Thu Sep 23, 2010 1:04 am
Forum: Sentence Correction
Topic: knewton sc vr 2
Replies: 7
Views: 1702

Hi all!

The task here is to avoid unnecessary calculations.
We don't need information about radius of the well.
We simply need to use formula for speed, which is v = s/t.
s = ?
v = 54 kmph
t = 5min = 5/60 h
s = v*t = 54*5/60 = 4.5 km
Therefore, correct answer is 3.

Hope it helps!
Best,
Maciek

by Maciek

Wed Sep 22, 2010 6:28 am
Forum: Problem Solving
Topic: a question from gmat.............
Replies: 3
Views: 1413

Hi all! IMO E x > 0.65(y + z) (1) x > 1.2y We don't know anything about z. Statement (1) ALONE is INSUFFICIENT We should eliminate answer choices A & D. (2) z/y = 45/52 x > 0.65(y + 45/52y) x > 0.65* 97/52y x > 65*97/5200y x > 6305/5200y Statement (2) ALONE is INSUFFICIENT We should eliminate an...

by Maciek

Wed Sep 22, 2010 2:29 am
Forum: Data Sufficiency
Topic: Interesting GMATFix Problem-19
Replies: 3
Views: 1161

Hi Aman! IMO C If the terrorist hears the two shots at shorter interval than a S.W.A.T. Team does, the S.W.A.T. Team is faster. It is because the distance between the S.W.A.T. Team and terrorist became shorter. By how much the S.W.A.T. Team is faster? x = (1 - 330/335)*1188 = (1 - 66/67)*1188 = 1188...

by Maciek

Wed Sep 22, 2010 2:04 am
Forum: Problem Solving
Topic: S.W.A.T
Replies: 5
Views: 1630

Hi Aman, There are two trains A and B. The length of the route is 390 km. The ratio of speed of the two trains is 6 : 7. Therefore, the trains A and B should exchange their speeds after travelling 6/(6+7)*390 km and 7/(6+7)*390 km , respectively. The answer should be either 180 km or 210 km. Correct...

by Maciek

Wed Sep 22, 2010 1:49 am
Forum: Problem Solving
Topic: Destination
Replies: 2
Views: 1093

Venmic, There are following issues here: subject-verb agreement , idiom and parallelism . 1) subject-verb agreement [/color] "computer" is a singual noun, so the verb must be also singular. We should eliminate answer choice A. 2) correct idiom is "as many as" We should eliminate ...

by Maciek

Wed Sep 22, 2010 12:49 am
Forum: Sentence Correction
Topic: WAS/WERE
Replies: 3
Views: 1624

IMO D There are following issues here: parallelism, logic of the sentence and idioms 1) parallelism The word “and� connects two sentences, which must be parallel. A: "To Henry David Thoreau..., and he" is not parallel B: "For Henry David Thoreau..., and he" is not parallel E:...

by Maciek

Wed Sep 22, 2010 12:09 am
Forum: Sentence Correction
Topic: To Henry David Thoreau, harmony with nature
Replies: 7
Views: 2948

Hi!

IMO E

x^2*y > 0 ?
x^2*y > 0 if y > 0 and (x < 0 or x > 0 )

(1) y^2 > 0
Therefore, y < 0 or y > 0
Statement (1) ALONE is INSUFFICIENT
(2) x = 2
Statement (2) ALONE is INSUFFICIENT
Statements (1) & (2) BOTH are together INSUFFICIENT.

Hope it helps!
Best,
Maciek

by Maciek

Fri Sep 17, 2010 3:25 pm
Forum: Data Sufficiency
Topic: positive/negative
Replies: 7
Views: 1447

Hi all! IMO E There are two main issues here: idiom and conditional. 1) correct idiom is "between X and Y" , where X and Y must be parallel. Therefore, "between competitions and race" is correct. We should eliminate answer choices A and B. 2) It should be correct type 0 condition...

by Maciek

Fri Sep 17, 2010 3:07 pm
Forum: Sentence Correction
Topic: knewton sc-4 hw 4
Replies: 2
Views: 1454

Hi! IMO C (1) p^3(1 - p^2) < 0 p^3(1 - p)(1 + p) < 0 /p^2 We can divide both sides by p^2 because p is not equal to 0 p(1 - p)(1 + p) < 0 -1 < p < 0 or p > 1 Statement (1) ALONE is NOT SUFFICIENT (2) p^2 - 1 < 0 (p - 1)(p + 1) < 0 -1 < p < 1 Statement (2) ALONE is NOT SUFFICIENT (1) & (2) (-1 < ...

by Maciek

Thu Sep 16, 2010 3:19 pm
Forum: Data Sufficiency
Topic: Is P negative???
Replies: 3
Views: 949

Hi all! IMO C Main issues here are 'if' versus 'whether' and verb tenses. 'Whether' should be used here. It is used to introduce a clause after a verb expressing or implying doubt or choice in order to indicate two or more alternatives. [1] Hence, we should eliminate answer choices A and B. Answers ...

by Maciek

Thu Sep 16, 2010 2:10 pm
Forum: Sentence Correction
Topic: Da Vinci first sketched the Mona Lisa
Replies: 9
Views: 1591

Nasir!
Your reasoning is correct.
8C5= the number of ways to select 5 union workers

The key word is 'at least'. Therefore, we need to add up combinations.

Either 4 union members work a shift or 5 union members work a shift.
Hence, we should add 8C5.

Hope it helps!
Best,
Maciek

by Maciek

Thu Sep 16, 2010 10:40 am
Forum: Problem Solving
Topic: Kaplan Combination Question
Replies: 3
Views: 1233